Translate

Saturday, June 7, 2014

Cấp số nguyên_Căn nguyên thủy

Bài toán 1 : Cho a,n,k là các số nguyên dương. Chứng minh rằng mọi ước nguyên tố của a^{2.6^{n}}-a^{6^{n}}+1 đều có dạng 6^{n+1}.k+1.
Lời giải :
Gọi p là ước nguyên tố của a^{2.6^{n}}-a^{6^{n}}+1. Ta có :
a^{2.6^{n}}-a^{6^{n}}+1\equiv 0\;(mod\;p)\Rightarrow a^{6^n}-a^{2.6^n}\equiv 1\;(mod\;p)\Rightarrow a^{6^n}(1-a^{6^n})\equiv 1\;(mod\;p)\Rightarrow -a^{2.6^n}.a^{6^n}\equiv 1(mod\;p)\Rightarrow a^{6^{n+1}}\equiv 1\;(mod\;p)\Rightarrow ord_p(a)\mid 6^{n+1}
Từ đây suy ra chỉ có thể là :
ord_p(a)\in \left \{ 2^k,3^k,6^k \right \},\;\forall k=\overline{1,n+1}
Nếu mà ord_p(a)\in \left \{ 2^k,3^k,6^t \right \},\;\forall k=\overline{1,n+1},\forall t=\overline{1,n} thì :
a^{2.6^{n}}-a^{6^{n}}+1\equiv 1\;(mod\;p)
Mâu thuẫn. Do vậy ord_p(a)=6^{n+1}
Bằng định lí Fermat nhỏ ta có ngay :
6^{n+1}=ord_p(a)\mid p-1
Điều phải chứng minh.
Bài toán 2 : Tìm số nguyên dương n sao cho n\mid 3^n-2^n.
Lời giải :
Hiển nhiên n=1 thỏa mãn. Xét n\geq 2, khi đó n có ước nguyên tố nhỏ nhất, gọi ước nguyên tố nhỏ nhất đó là p.
Gọi t\in \mathbb{N} là nghịch đảo của 2 modulo p, tức là 2t\equiv 1\;(mod\;p),1\leq t\leq p-1.
Ta có 3^{n}\equiv 2^{n}\;(mod\;p)\Rightarrow (3t)^{n}\equiv (2t)^{n}\equiv 1\;(mod\;p)\Rightarrow ord_p(3t)\mid n\;\;(1)
Nếu p=3 thì 3\mid 3\Rightarrow 3\mid 3^{n}-2^{n}\Rightarrow 3\mid 2^n (vô lí). Vậy p\neq 3 và vì 1\leq t\leq p-1 nên gcd(t,p)=1\Rightarrow gcd(3t,p)=1.
Theo định lí Fermat nhỏ, ta có \left ( 3t \right )^{p-1}\equiv 1\;(mod\;p)\Rightarrow ord_p(3t)\mid p-1\;\;(2)
Từ (1)(2) suy ra ord_p(3t) có một ước nguyên tố r mà r\mid n và r\leq p-1<p. Điều này mâu thuẫn với tính nhỏ nhất của p. Suy ra ord_p(3t)=1\Rightarrow 3t\equiv 1\equiv 2t\;(mod\;p)\Rightarrow t\equiv 0\;(mod\;p)
Ta gặp điều mâu thuẫn.
Kết luận : Có duy nhất một số nguyên dương n thỏa đề là n=1.
Bài toán 3 : Cho  n là số nguyên dương thỏa mãn n\mid 2^{n}+5^{n}. Chứng minh rằng n chia hết cho 7.
Lời giải :
Dễ thấy n là số nguyên dương lẻ. Gọi p là ước nguyên tố bé nhất của n.
Gọi a là nghịch đảo của 2 modulo p. Khi đó thì 2a\equiv 1\;(mod\;p),gcd(2,a)=gcd(a,p)=1.
Ta có p\mid 2^{n}+5^{n}\Rightarrow p\mid (2a)^n+(5a)^n\Rightarrow (2a)^n+(5a)^{n}\equiv (5a)^n+1\;(mod\;p)\Rightarrow -(5a)^n=(-5a)^n\equiv 1\;(mod\;p)\Rightarrow ord_p(-5a)\mid n\;\;\;\;(1)
Dễ dàng thấy gcd(5a,p)=1 nên theo định lí Fermat nhỏ ta có (-5a)^{p-1}\equiv 1\;(mod\;p)\Rightarrow ord_p(-5a)\mid p-1\;\;\;\;(2)
Từ (1)(2) suy ra tồn tại một ước nguyên tố r của ord_p(-5a) mà r<p-1<p,r\mid n. Điều này mâu thuẫn với tính nhỏ nhất của p. Như vậy phải có ord_p(-5a)=1. Suy ra -5a\equiv 1\equiv 2a\;(mod\;p)\Rightarrow 7a\equiv 0\;(mod\;p)
Mà gcd(a,p)=1 nên p=7. Suy ra n chia hết cho 7.
Bài toán 3 (CĐT Olympic 30-4 môn toán 10 năm 2013-2014 THPT Chuyên Trần Hưng Đạo tỉnh Bình Thuận) 
 Tìm các số nguyên tố p,q thỏa mãn pq \mid p^p+q^q+1.
Lời giải :
Bổ đề : Cho các số nguyên tố p,q,r trong đó p lẻ và thỏa mãn p \mid q^r+1 thì khi đó 2r \mid p-1 hoặc p\mid q^2-1.
Bài toán :
Từ đề bài ta có p^{p}+1\equiv 0\;(mod\;q) và q^{q}+1\equiv 0\;(mod\;p)
Xét p=2 ta có 4+1\equiv 0\;(mod\;q)\Rightarrow q=5, thử lại cặp (2,5) thỏa mãn. Tương tự cặp (5,2) cũng thỏa mãn.
Xét các số nguyên tố p,q đều lẻ.
Vì q\mid p^p+1 nên áp dụng bổ đề ta có 2p\mid q-1 hoặc q\mid p^2-1.
Nếu 2p\mid q-1\Rightarrow q\equiv 1\;(mod\;p)\Rightarrow 0\equiv q^{q}+1\equiv 2\;(mod\;p)\Rightarrow p=2 (loại)
Nếu q\mid p^2-1 thì q\mid p-1 hoặc q\mid p+1. Lập luận như trên ta chỉ ra rằng q\mid p-1 là vô lí nên phải có q\mid p+1. Tuy nhiên thì p+1 chẵn và gcd(q,2)=1 nên ta có 2q\mid p+1\Rightarrow p+1\geq 2q.
Hoàn toàn tương tự ta có q+1\geq 2p. Suy ra p+q+2\geq 2(p+q)\Rightarrow p+q\leq 2 và điều này thì vô lí
Kết luận : \boxed{(p,q)=(2,5),(5,2)}
Bài toán 4 : Tìm số nguyên dương n thỏa mãn n\mid 2^n-1.
Lời giải :
Ta thấy n=1 thỏa mãn. Xét n>1. Gọi p là ước nguyên tố bé nhất của n.
Theo đề bài ta có 2^{n}\equiv 1\;(mod\;p)\Rightarrow ord_p(2)\mid n.
Theo định lí Fermat nhỏ thì 2^{p-1}\equiv 1\;(mod\;p)\Rightarrow ord_p(2)\mid p-1\Rightarrow p>p-1>ord_p(2)
Ta gọi q là ước nguyên tố của ord_p(2), ta thấy q\mid n và p>q. Điều này mâu thuẫn vì p là ước nguyên tố bé nhất của n. Trường hợp này không tìm được n thỏa đề.
Kết luận : Có duy nhất số nguyên dương thỏa mãn đề bài là \boxed{n=1}
Bài toán 5 (USA TST 2003): Tìm các số nguyên tố p,q,r thỏa mãn đồng thời p\mid q^r+1\;,\;q\mid r^p+1\;,\;r\mid p^q+1.
Lời giải :
Bổ đề : Cho các số nguyên tố p,q,r trong đó p lẻ và thỏa mãn p\mid q^r+1 thì khi đó 2r\mid p-1 hoặc p\mid q^2-1
Xem chứng minh bổ đề tại đây
Trở lại bài toán.
Nhận thấy rằng các số nguyên tố p,q,r phải phân biệt.
  • Trường hợp 1 : Xét các số nguyên tố p,q,r đều lẻ.
Theo bổ đề ta có 2r\mid p-1 hoặc p\mid q^2-1.
Nếu 2r\mid p-1\Rightarrow p\equiv 1\;(mod\;r)\Rightarrow 0\equiv p^{q}+1\equiv 2\;(mod\;r)\Rightarrow r=2 (loại)
Do vậy phải có p\mid q^2-1\Rightarrow p\mid q-1\;\;\vee p\mid q+1.
Nếu p\mid q-1\Rightarrow q\equiv 1\;(mod\;p)\Rightarrow 0\equiv q^{r}+1\equiv 2\;(mod\;p)\Rightarrow p=2 (loại)
Suy ra p\mid q+1 mà q+1 chẵn và gcd(2,p)=1 nên 2p\mid q+1, từ đó q+1\geq 2p
Hoàn toàn tương tự ta được r+1\geq 2q và p+1\geq 2r.
Như vậy p+q+r+3\geq 2(p+q+r)\Leftrightarrow p+q+r\leq 3 và đây là điều vô lí.
  • Trường hợp 2 : Trong các số p,q,r có ít nhất một số chẵn. Gỉa sử r=2.
Khi đó giả thiết trở thành q\mid 2^p+1 và p\mid q^2+1.
Cũng theo bổ đề trên thì ta được q\mid r^{2}-1 hoặc 2p\mid q-1. Nếu mà 2p\mid q-1 thì q\equiv 1\;(mod\;p)\Rightarrow 0\equiv q^{2}+1\equiv 2\;(mod\;p)\Rightarrow p=2 (loại vì p,q,r phải phân biệt)
Như vậy có q\mid r^2-1=2^2-1=3\Rightarrow q=3. Từ đó p\mid q^2+1=3^2+1=10\Rightarrow p=5
Bộ số (p,q,r)=(5,3,2) thoả mãn.
Kết luận : \boxed{(p,q,r)=(5,3,2),(2,5,3),(3,2,5)}
Bài toán 6 : Cho số nguyên dương n lớn hơn 1 và thỏa mãn n\mid 3^n-1. Chứng minh rằng n là số chẵn.
Lời giải :
Gọi p là ước nguyên tố bé nhất của n
Theo giả thiết thì 3^{n}\equiv 1\;(mod\;p)\Rightarrow ord_p(3)\mid n\;\;\;(1)
Hiển nhiên p\neq 3 vì nếu vậy thì 3\mid 3^{n}-1 (vô lí). Khi đó theo định lí Fermat nhỏ ta có 3^{p-1}\equiv 1\;(mod\;p)\Rightarrow ord_p(3)\mid p-1\Rightarrow p>p-1>ord_p(3)\;\;\;(2).
Gọi q là một ước nguyên tố của ord_p(3) thì theo (1)q là một ước nguyên tố của n nhưng theo (2) thì p>q. Điều này mâu thuẫn với tính nhỏ nhất của p.
Suy ra ord_p(3)=1. Khi đó 3\equiv 1\;(mod\;p)\Rightarrow p=2. Suy ra n chẵn. Đây là điều phải chứng minh.
Tổng quát bài toán : Cho số nguyên tố p sao cho tồn tại số nguyên dương n sao cho n\mid (p+1)^n-1. Chứng minh rằng n chia hết cho p.
Bài toán 7 (IMO Shortlist 2006)
Tìm các số nguyên dương x,y thỏa mãn phương trình \dfrac{x^7-1}{x-1}=y^5-1.
Lời giải :
Bổ đề : Cho các số nguyên dương x,m (x>1) và số nguyên tố p thỏa mãn m\mid \dfrac{x^p-1}{x-1}. Khi đó thì m\equiv 0,1\;\;(mod\;p)
Chứng minh bổ đề :Gọi r là một ước nguyên tố của m.
Từ đề bài ta có
m\mid x^{p}-1\Rightarrow x^{p}\equiv 1\;(mod\;m)\Rightarrow x^{p}\equiv 1\;(mod\;r)\Rightarrow ord_x(r)\mid p\Rightarrow ord_x(r)\in \left \{ 1,p \right \}
Nếu ord_x(r)=1 thì x\equiv 1\;(mod\;r)\Rightarrow \dfrac{x^{p}-1}{x-1}=x^{p-1}+x^{p-2}+...+x+1\equiv \underset{p}{\underbrace{1+1+...+1}}=p\;(mod\;r)
Mà \dfrac{x^p-1}{x-1}\equiv 0\;(mod\;r)\Rightarrow p\equiv 0\;(mod\;r)\Rightarrow p=r\Rightarrow m\equiv 0\;(mod\;p)
Nếu ord_x(r)=p, hiển nhiên r\nmid x vì nếu r\mid x thì \dfrac{x^p-1}{x-1}\equiv 1\;(mod\;r) và điều này trái giả thiết.
Do đó áp dụng định lí Fermat nhỏ thì x^{r-1}\equiv 1\;(mod\;r), suy ra ord_x(r)\mid r-1\Rightarrow p\mid r-1\Rightarrow r\equiv 1\;(mod\;p)\Rightarrow m\equiv 1\;(mod\;p)
Như vậy ta có m\equiv 0,1\;(mod\;p). Bổ đề được chứng minh.
Trở lại bài toán :
Ta viết phương trình dưới dạng : \dfrac{x^7-1}{x-1}=(y-1)(y^5+y^4+y^3+y^2+y+1)
Áp dụng bổ đề trên thì ta có \left\{\begin{matrix} y-1\equiv 0,1\;(mod\;7) & (1) & \\ y^4+y^3+y^2+y+1\equiv 0,1\;(mod\;7) & (2) & \end{matrix}\right.
Nhưng từ (1) ta có y\equiv 1,2\;(mod\;7)\Rightarrow y^4+y^3+y^2+y+1\equiv 5,3\;(mod\;7). Mâu thuẫn với (2).
Kết luận : Không tồn tại các số nguyên dương x,y thỏa mãn đề bài.

No comments: